Wie nah ist eine ungerade Zweierpotenz an einem perfekten Quadrat?

Lassen N 1 eine ungerade natürliche Zahl sein. Definieren

F ( N ) = Mindest { | k | | k + 2 N ist ein Quadrat , k Z } .

Das ist F ( N ) misst, wie nahe die Leistung ist 2 N zu einem perfekten Quadrat.

Ich schätze, dass dieser Begriff irgendwo untersucht wurde, aber ich konnte ihn nicht naiv bei Google finden.

Frage: (etwas weich)

Hat diese Funktion einen bekannten Namen in der Literatur? Wurde das irgendwo studiert? Gibt es dafür eine Formel in geschlossener Form oder zumindest einige nette Untergrenzen für seine Werte?

Das ist A238454 ... einfach zu berechnen, aber ich sehe keine offensichtlichen Verbindungen zu bekannten Sequenzen.
@lulu Dies ist nicht genau A238454, das das nächstgrößere Quadrat auflistet . OP fragt nach dem nächsten Quadrat für k Z , Berichterstattung F ( N ) = | k | . So für N = 7 , A238454 Listen 16 , Sein 144 128 . Aber 121 128 gibt k = 7 , | k | = 7
@KeithBackman Ah, danke. Ich habe mich verlesen und nur das größere in Betracht gezogen.

Antworten (3)

Uneffektiv kann man das zeigen, gegeben ϵ > 0 , gibt es eine positive Konstante C ( ϵ ) so dass

F ( N ) C ( ϵ ) 2 ( 1 / 2 ϵ ) N .
Dies folgt aus der P -adische Version des Satzes von Roth, bewiesen von Ridout, und stellt den wahren Sachverhalt dar. Ich vermute, dass es sehr schwierig wäre, dies effektiv zu machen, und alles, was einer geschlossenen Form ähnelt, ist zu viel zu hoffen.

In Bezug auf explizite untere Schranken kann man das beweisen

F ( N ) > 2 0,26 N ,
es sei denn N { 3 , 15 } . Dies kann in einer alten Arbeit von Bauer et al. gefunden werden; Der Beweis verwendet die Pade-Annäherung an die Binomialfunktion. Es ist unwahrscheinlich, dass man mit elementaren Methoden eine große Grenze erreichen kann.

N = 15 ist besonders interessant, wie 2 15 181 2 = 7 .

In Anbetracht möglicher Entfernung k zwischen einer ungeraden Potenz von 2 und dem nächsten ganzzahligen Quadrat (siehe A236564 ), kann es hilfreich sein, die Frage allgemeiner zu betrachten und zunächst einige Unmöglichkeiten zu bemerken.

Für eine ungerade N T H Kraft von 2 , 2 N 2 ( Mod 3 ) . Aber für jede ganze Zahl M , M 2 1 oder 0 ( Mod 3 ) . Somit 3 kann sich nicht teilen

| 2 N M 2 |

Ebenso, da ungerade Potenzen von 2 2 , 3 ( Mod 5 ) , Aber M 2 1 , 4 , 0 ( Mod 5 ) , Dann 5 wird sich nicht teilen

| 2 N M 2 |

Aber seit 2 N 2 , 1 , 4 ( Mod 7 ) , während M 2 1 , 4 , 2 , 0 ( Mod 7 ) , Dann 7 wird sich teilen | 2 N M 2 | wann immer

2 N M 2 ( Mod 7 )
Z.B
2 7 11 2 2 ( Mod 7 )
Nach 1 Und 4 , Dann, 7 ist die nächste Potenz, die ein ganzzahliges Quadrat zu einer ungeraden Potenz haben kann 2 .

Die gleiche Methode zeigt, dass beides nicht der Fall ist 11 noch 13 teilt

| 2 N M 2 |
seit
2 N 2 , 8 , 10 , 7 , 6 ( Mod 11 )
Aber
M 2 1 , 4 , 5 , 9 , 3 , 0 ( Mod 11 )
Und wieder,
2 N 2 , 8 , 6 , 11 , 5 , 7 ( Mod 13 )
Aber
M 2 1 , 4 , 9 , 3 , 12 , 10 , 0 ( Mod 13 )
Aber seit
2 N 2 , 8 , 15 , 9 ( Mod 17 )
Und
M 2 1 , 4 , 9 , 16 , 8 , 2 , 15 , 13 , 0 ( Mod 17 )
Dann 17 wird sich teilen | 2 N M 2 | wann immer
2 N M 2 ( Mod 17 )
z.B
2 9 23 2 2 ( Mod 17 )
Ähnlich, 19 ausgeschlossen werden kann und 23 ist die nächstkleinere ungerade Primzahl dazwischen 2 N Und M 2 :
2 11 45 2 1 ( Mod 23 )

Im folgenden Array 2 N liegt zwischen aufeinanderfolgenden ganzzahligen Quadraten M 2 , ( M + 1 ) 2 , mit k M = 2 N M 2 , k M + 1 = ( M + 1 ) 2 2 N . Das kleinere k für jede 2 N ist fett gedruckt .

M 2 k M 2 N k ( M + 1 ) ( M + 1 ) 2 1 2 1 2 1 2 2 2 2 2 2 2 2 3 1 3 2 5 2 7 2 5 2 2 6 2 11 2 7 2 7 2 4 12 2 22 2 2 2 7 2 9 17 23 2 45 2 23 2 11 2 2 17 46 2 90 2 2 2 23 2 13 89 91 2 181 2 7 2 15 2 2 89 182 2 362 2 2 2 7 2 17 17 41 363 2 724 2 2 4 7 2 19 17 191 725 2 1448 2 2 6 7 2 21 31 79 1449 2 2896 2 2 8 7 2 23 4001 2897 2 5792 2 2 10 7 2 25 7 631 5793 2 11585 2 5503 2 27 2 2 7 631 11586 2 23170 2 2 2 5503 2 29 24329 23171 2 46340 2 2 4 5503 2 31 41 113 46341 2 92681 2 7 23833 2 33 2 2 41 113 92682 2 185363 2 17 73 239 2 35 2 4 41 113 185364 2 370727 2 31 2 463 2 37 2 6 41 113 370728 2 741455 2 7 42409 2 39 2 8 41 113 741456 2 1482910 2 2 2 7 42409 2 41 79 22511 1482911 2 2965820 2 2 4 7 42409 2 43 761 1553 2965821 2 5931641 2 191 37361 2 45 2 2 761 1553 5931642 2 11863283 2 7 2 17 5783 2 47 2 4 761 1553 11863284 2 23726566 2 2 2 7 2 17 5783 2 49 7 23 31 5647 23726567 2 47453132 2 2 4 7 2 17 5783 2 51 17830441 47453133 2 94906265 2 118490767 2 53 2 2 17830441 94906266 2 189812531 2 41 2300927 2 55 2 4 17830441 189812532 2 379625062 2 2 2 41 2300927 2 57 7 359 151969 379625063 2 759250124 2 2 4 41 2300927 2 59 9092137 759250125 2 1518500249 2 47 137 466009 2 61 2 2 9092137 1518500250 2

Für N 61 , k besteht nur aus geraden Potenzen von 2 (seit 1 = 2 0 ) und Primzahlen P { 1 , 2 , 7 } ( Mod 8 ) :

2 , 7 , 17 , 23 , 31 , 41 , 47 , 73 , 79 , 89 , 113 , 137 , 191 , 239 , 359 , 463 , 631 , 761 , 1553 , 4001 , 5503 , 5647 , 5783 , 22511 , 23833 , 24329 , 37361 , 42409 , 151 969 , 2 300 927 , 9 092 137 , 17 830 441 , 118 490 767

(Vergleiche A038873 )

Von den elf solchen ungeraden Primzahlen < 101 , neun erscheinen als Faktoren von k im Beispiel oben. Und von den restlichen zwei:

k M = 6 925 661 896 845 871 = 97 3407 20 956 435 649
für N = 113 , Und
k M + 1 = 704 428 023 733 764 953 = 7 17 23 31 41 71 21377 133 417
für N = 117 .

Es scheint also, dass außerdem 2 und seine geraden Potenzen, alle und nur alle ungeraden Primzahlen P 1 , 7 ( Mod 8 ) Faktoren sein können

k = | 2 N M 2 |
für M 2 allgemein und damit auch für M 2 am nächsten 2 N .

Andere Beobachtungen/Vermutungen

Es ist natürlich zu fragen, ob, wie 7 , einige zweistellige Primzahlen, zB 17 , 23 , 89 , oder vielleicht eine zweistellige zusammengesetzte Zahl, z 2 2 7 = 28 , kann als niedrigerer Wert von ein zweites Mal auftreten k für einige N . Ganz klar jeder 2 N ist das Vierfache des vorhergehenden. Und wann immer M oder M + 1 ist dann das Doppelte seines Vorgängers M 2 oder ( M + 1 ) 2 wird auch sein Vorgänger vervierfachen, und folglich wird auch der Wert von k . Z.B k M vervierfacht fünfmal für N = 15 25 . Nochmal, k ( M + 1 ) , vervierfacht sich viermal für N = 31 39 .

Wenn jeder M Und M + 1 waren doppelt so groß wie die vorhergehende, k würde sich mit jeder Erhöhung für immer vervierfachen N : kein Wert von k könnte sogar einmal wieder auftreten, wie 1 Und 7 Tun. Aber natürlich seitdem M , M + 1 sind für alle gegebenen Werte von entgegengesetzter Parität N , sie können niemals beide das Doppelte ihres Vorgängers sein: vielmehr ungefähr so ​​oft, wie es sich verdoppelt, M erhöht sich um doppelt-plus-eins, und M + 1 durch Doppel-Minus-Eins (siehe Tabelle oben). Und dann, anstatt zu vervierfachen, k kann sogar noch weniger als die vorherige werden k , wie es auch passiert k M oder k M + 1 für

N = 3 , 11 , 15 , 21 , 27 , 31 , 31 , 39 , 43 , 47 , 51 , 55 , 59
in der Tabelle oben.

Andererseits betrachtet man nur kleineres k : für ungerade N 999 (Vgl. A236564, Links), k für 2 N nie überschreitet k für 2 N + 2 um mehr als zwei Ziffern. Und selbst dies kommt nur sechsmal vor, z

N = 59 , 527 , 669 , 695 , 905 , 973
Nach dieser konservativen Regel würden wir also keine neuen oder wiederkehrenden zweistelligen Zahlen erwarten k nach den letzten vier Ziffern k = 5503 für N = 27 , Komposit machen k = 2 2 7 = 28 für N = 17 die letzten zwei Ziffern k . Ebenso scheint es dreistellig zu sein k muss nach der letzten fünf Ziffer enden k = 2 4 41 113 = 74128 für N = 35 , machen 2 6 7 = 448 für N = 21 die letzten drei Ziffern k . Es scheint keine dreistellige Primzahl zu geben k . Nach der letzten sechsstelligen k = 7 42409 = 296863 , für N = 39 , sollte es nicht mehr vierstellig sein k : 5503 ist die letzte und die einzige vierstellige Primzahl k .

Es gibt die Vermutung von Catalan und den Satz von Michailescu, die dies besagen Y P = 1 + X Q hat nur eine Lösung für X , Y > 0 . Diese Zahlen sind 3 2 = 1 + 2 3 . Seit 2 2 N + 1 kann kein perfektes Quadrat sein, 0 kann keine Lösung sein k , der geht k = 1 , wie oben, als absolut nächste Lösung. Ich nehme an, relativ gesehen wird es viele seltsame Potenzen von geben 2 so dass 2 2 N + 1 / Y P ∣< 9 / 8 = 1 ± ε wo immer höhere ungerade Potenzen von 2 reduziert ε auf eine beliebig niedrige Zahl.